Đến nội dung

Gianghg8910

Gianghg8910

Đăng ký: 17-07-2018
Offline Đăng nhập: 08-08-2019 - 13:25
-----

Trại hè hùng vương 2019

31-07-2019 - 10:19

Mọi người có thể giúp mình câu 3(đa thức) với câu 5(tổ hợp) được không ạ


SỐ HỌC

24-07-2019 - 10:53

CHo mình hỏi bài này tại sao theo FERMAT thì q là ước của 39.

FERMAT này là định lý nào ạ.

Với lại tại sao 5^q-2^q đồng dư với 5-2(mod q)

Mong các cao thủ giúp cho.Mình cảm ơn


Bất biến

21-07-2019 - 17:27

Các bạn có thể giải thích giùm mình chỗ khoanh đen được ko ạ?

Tại sao bộ trạng thái 4 lại nhận được từ trạng thái 2 bằng các bình phương??

Mong các cao thủ giải đáp.


Hình học phẳng

21-07-2019 - 09:50

Mọi người ơi hãy cùng thảo luận để đưa ra lời giải cho bài sau

 


Trại hè Phương Nam

19-07-2019 - 17:56

Mọi người hãy cùng thảo luận câu 3 với câu 4(2 câu này mình chưa ra)

Mong mọi người tích cực thảo luận